Lorentz invariant integral measure

In summary, the conversation was about the Lorentz invariant integration measure and how it can be used to show that d3p/E is a Lorentz scalar. The person asking the question wanted to find another way to show that d(p^2) is Lorentz invariant and tried using the Lorentz transformed equation. However, their attempt was incorrect because E is not the same as p^0. The expert then provided a partial solution, using the example of a sphere in 3-dimensional Euclidean space, to illustrate the concept of rotation invariance and its relation to Lorentz invariance. They also mentioned that a more direct proof without using a delta function could be done, but it would be more complicated
  • #1
DOTDO
7
0
Hi

I'm studying electron-muon scattering

and now considering the Lorentz invariant integration measure.

The textbook introduced it, which use dirac delta function to show that d3p/E is a Lorentz scalar.

I understood it but I wanted to find other way and tried like this:
20150923_154737-1.jpg


I need a hint on the underlined equation. The primed one is lorentz transformed one.

How can I show that d(p^2) is Lorentz invariant?

Just calculate it hard ?
 
Last edited:
Physics news on Phys.org
  • #2
Your attempt has a mistake because ##E## is not the same as ##p^0##. The four variables ##p^0,p^1,p^2,p^3\equiv (p^0,{\bf p})## are 4 independent variables, but ##E=\sqrt{{\bf p}^2+m^2}## is not independent of ##{\bf p}##. Hence, your conclusion that ##d{\bf p}^2## should be Lorentz invariant is wrong.

The measures ##d^4p=dp^0 d^3p## and ##\frac{d^3p}{E}## are Lorentz invariant, but ##dE d^3p## and ##\frac{d^3p}{p^0}## are not.

If you are still confused, try first to solve the following problem. Consider 3-dimensional Euclidean space with Cartesian coordinates ##x,y,z##. In this space, consider the sphere defined by ##x^2+y^2+z^2=R^2##, which is a 2-dimensional object invariant under 3-dimensional rotations. Find the measure of area ##dA## on the upper half of the sphere, in the form ##dA=f(x,y)dxdy##. Argue that this measure is rotation-invariant simply because the sphere is rotation-invariant. Find ##f(x,y)## explicitly in two ways: (i) geometrically and (ii) by starting from volume element ##dV=dxdydz## and using a ##\delta##-function trick. Can you see the relation with the problem of Lorentz-invariant measure above? In particular, can you see the analogy between ##R## and ##m##?

Partial solution: You should get $$f(x,y)=\frac{R}{\sqrt{R^2-x^2-y^2}}.$$Is it correct to write the right-hand side as ##R/z##?
 
Last edited:
  • #3
Demystifier said:
(ii) by starting from volume element dV=dxdydz and using a δ-function trick.
Since I have nothing more important to do at the moment, let me do this explicitly. In spherical coordinates we have
$$dV=dA dr$$
where ##dA## is the area element of the sphere. Since ##dr\delta(r-R)=1##, we can use this to write
$$dA=dV\, \delta(r-R). \;\;\; (1)$$
We also have
$$\delta(R^2-r^2)=\delta((R-r)(R+r))=\frac{\delta(R-r)}{2R}$$
so (1) can be written as
$$dA=dV \, 2R\delta(R^2-r^2) \;\;\; (2)$$
Clearly, (2) is rotation invariant. In Cartesian coordinates this can be written as
$$dA=dxdydz \, 2R\delta(R^2-x^2-y^2-z^2) \;\;\; (3)$$
Introducing the function
$$w(x,y)=\sqrt{R^2-x^2-y^2}$$
which is non-negative on the sphere of radius ##R##, (3) can be written as
$$dA=dxdydz \, 2R\delta(w^2-z^2)=dxdydz \, 2R\delta((w-z)(w+z)) \;\;\; (4)$$
For the upper half of the sphere we have ##z>0##, so for the upper half (4) can be written as
$$dA=dxdydz \, 2R \frac{\delta(w-z)}{2w}=dxdy \frac{R}{w}. \;\;\; (5)$$

Since this is rotation invariant and since ##R## is a constant, it follows that the measure
$$\frac{dxdy}{w}$$
is rotation invariant. This is analogous to the fact that the measure
$$\frac{d^3p}{\omega}$$
with ##\omega=\sqrt{m^2+{\bf p}^2}## is Lorentz invariant. The differences are that (i) for Lorentz invariance we have one dimension more, and (ii) some signs are different because Minkowski metric is different from the Euclid metric.
 
Last edited:
  • Like
Likes haushofer
  • #4
Isn't this a bit too complicated? Usually the derivation starts from the fact that ##\mathrm{d}^4 p## is invariant under proper Lorentz transformations. Then the on-shell ##\delta##-distribution, ##\delta(p^2-m^2)=\delta[(p^0)^2-E^2]##, where ##E^2=\vec{p}^2+m^2##. Finally ##\Theta(p^0)## is a Lorentz invariant under proper orthochronous Lorentz transformations and thus finally
$$\mathrm{d}^4 p \Theta(p^0) \delta(p^2-m^2)=\mathrm{d} p^0 \mathrm{d}^3 \vec{p} \frac{1}{2E} \delta(p^0-E)$$
is Lorentz invariant. Thus if you have a function ##f(p)## with ##p^0=E## you can write
$$\int \mathrm{d}^4 p \Theta(p^0) \delta(p^2-m^2)=\int \mathrm{d}^3 \vec{p} \frac{1}{2E} f(p^0=E,\vec{p}).$$
If ##f(p)=f(p^0,\vec{p})## is a scalar field, then for the on-shell function ##f(p^0=E,\vec{p})## the 3D integral with the measure ##\mathrm{d}^3 \vec{p}/E## is a scalar.
 
  • #5
vanhees71 said:
Isn't this a bit too complicated?
Yes, but he already indicated that he understands the standard proof of Lorentz invariance with the use of a delta function. He wanted a more direct proof without a delta function, and in his attempt he failed. I wanted to explain him what exactly was wrong with his attempt. In addition, I also thought that it would be easier to undertstand this in a case which can be more easily visualized, i.e. where 4-dimensional Lorentz invariance is replaced with 3-dimensional rotational invariance. While the calculation turned out to be longer, I thought that the resulting geometric idea would be easier to understand intuitively.
 
  • Like
Likes vanhees71
  • #6
Demystifier said:
Your attempt has a mistake because E is not the same as ##p^0##. The four variables ## p^0,p^1,p^2,p^3\equiv (p^0,{\bf p}) ## are 4 independent variables, but ##E=\sqrt{{\bf p}^2+m^2}## is not independent of p.
I don't understand this part.
1) The 4-momentum is always defined to be ## p^\mu=(E,\bf p) ## in a particular frame. Now you say ## p^0 ## is not the same as E. So what is it?
2) You say that different components of the 4-momentum are independent but what about the restriction ## p^2=-m^2 ## on any 4-momentum?
 
  • #7
Shyan said:
I don't understand this part.
1) The 4-momentum is always defined to be ## p^\mu=(E,\bf p) ## in a particular frame. Now you say ## p^0 ## is not the same as E. So what is it?
2) You say that different components of the 4-momentum are independent but what about the restriction ## p^2=-m^2 ## on any 4-momentum?
Since ##E## is defined as ##\sqrt{{\bf p}^2+m^2}##, these two questions are really the same. To answer them, it should be enough to give one example where ##p^{\mu}## is not equal to ##(E,\bf p)##. So here is one example: the 4-momentum integration in a Feynman diagram with a loop.

If you are still confused, think about doing the exercise proposed in #2.
 
  • #8
Demystifier said:
Since ##E## is defined as ##\sqrt{{\bf p}^2+m^2}##, these two questions are really the same. To answer them, it should be enough to give one example where ##p^{\mu}## is not equal to ##(E,\bf p)##. So here is one example: the 4-momentum integration in a Feynman diagram with a loop.

If you are still confused, think about doing the exercise proposed in #2.

I think I understand it now. It seems that the right way of thinking about 4-momentums is that there is a 4-dimensional momentum space and particles move on ## p^2=-m^2 ## hypersurfaces in this space. I was missing this viewpoint. thanks.
 
  • Like
Likes vanhees71 and Demystifier

Related to Lorentz invariant integral measure

1. What is a Lorentz invariant integral measure?

A Lorentz invariant integral measure is a mathematical concept used in relativistic physics to describe the integral of a function over spacetime that remains unchanged under Lorentz transformations. It ensures that physical laws and measurements are consistent across different reference frames.

2. How is a Lorentz invariant integral measure calculated?

A Lorentz invariant integral measure is calculated by taking into account the Lorentz transformation properties of the function being integrated. This involves using the Lorentz transformation equations to transform the function and its variables, and then integrating over the transformed variables.

3. What is the significance of Lorentz invariant integral measure in physics?

Lorentz invariant integral measure is crucial in relativistic physics as it allows for the consistent description of physical phenomena across different inertial frames of reference. It is a fundamental aspect of special relativity and is used in various calculations and equations, such as in the formulation of Einstein's famous equation E=mc².

4. Can Lorentz invariant integral measure be applied to any function?

Lorentz invariant integral measure can be applied to any function as long as it is properly transformed in accordance with Lorentz transformation properties. This includes scalar, vector, and tensor functions.

5. Is Lorentz invariant integral measure only used in relativistic physics?

No, Lorentz invariant integral measure can also be used in other areas of physics, such as quantum field theory and particle physics, where the principles of special relativity are applied. It is a crucial concept in these fields for maintaining consistency and accuracy in measurements and calculations.

Similar threads

  • Special and General Relativity
3
Replies
101
Views
3K
  • Special and General Relativity
Replies
9
Views
1K
  • Special and General Relativity
Replies
1
Views
1K
  • Special and General Relativity
Replies
8
Views
5K
  • Special and General Relativity
Replies
11
Views
4K
  • Special and General Relativity
Replies
5
Views
991
  • Special and General Relativity
Replies
4
Views
2K
  • Special and General Relativity
Replies
5
Views
1K
  • Special and General Relativity
Replies
5
Views
1K
  • Special and General Relativity
Replies
32
Views
4K
Back
Top